2021 AMC 12B Problems/Problem 5

Revision as of 18:41, 11 February 2021 by Sugar rush (talk | contribs)

Problem

The point $P(a,b)$ in the $xy$-plane is first rotated counterclockwise by $90\deg$ around the point $(1,5)$ and then reflected about the line $y = -x$. The image of $P$ after these two transformations is at $(-6,3)$. What is $b - a ?$

$\textbf{(A)} ~1 \qquad\textbf{(B)} ~3 \qquad\textbf{(C)} ~5 \qquad\textbf{(D)} ~7 \qquad\textbf{(E)} ~9$

Solution

$\boxed{\textbf{(D)} ~7}$